The figure below shows a top view of a bar that can slide on two frictionless rails. The resistor is R = 6.60 N, and a 2.50-T magnetic field is directed perpendicularly downward, into the page. Let l = 1.20 m. R X x x x x x x X xx X X X X X x x x x x x x x x x xxx x xx X x Bin X *F x x X x * X app (a) Calculate the applied force required to move the bar to the right at a constant speed of 2.50 m/s. N (to the right) (b) At what rate is energy delivered to the resistor? W

College Physics
10th Edition
ISBN:9781285737027
Author:Raymond A. Serway, Chris Vuille
Publisher:Raymond A. Serway, Chris Vuille
Chapter20: Induced Voltages And Inductance
Section: Chapter Questions
Problem 56AP
icon
Related questions
Question

The figure below shows a top view of a bar that can slide on two frictionless rails. The resistor is 

R = 6.60 Ω,

 and a 2.50-T magnetic field is directed perpendicularly downward, into the page. Let ℓ = 1.20 m.

A vertical bar and two parallel horizontal rails lie in the plane of the page, in a region of uniform magnetic field, vector Bin, pointing into the page. The parallel rails run from left to right, with one a distance ℓ above the other. The left ends of the rails are connected by a vertical wire containing a resistor R. The vertical bar lies across the rails to the right of the wire. Force vector Fapp points from the bar toward the right.
(a) Calculate the applied force required to move the bar to the right at a constant speed of 2.50 m/s.
 N (to the right)

(b) At what rate is energy delivered to the resistor?
 W
The figure below shows a top view of a bar that can slide on two frictionless rails. The resistor is R = 6.60 , and a 2.50-T
magnetic field is directed perpendicularly downward, into the page. Let l = 1.20 m.
www
R
X * X x
* X
X
X
X X
xx
x 1
x
е
xxxx
X
X
xxxx
x
X x
xxxxX
>
x
X X X x X
X
>
X
X
x
x
xxxx
X
xx
x →
x Bin
xx
*F
xx
* *
app
i
(a) Calculate the applied force required to move the bar to the right at a constant speed of 2.50 m/s.
N (to the right)
(b) At what rate is energy delivered to the resistor?
W
Transcribed Image Text:The figure below shows a top view of a bar that can slide on two frictionless rails. The resistor is R = 6.60 , and a 2.50-T magnetic field is directed perpendicularly downward, into the page. Let l = 1.20 m. www R X * X x * X X X X X xx x 1 x е xxxx X X xxxx x X x xxxxX > x X X X x X X > X X x x xxxx X xx x → x Bin xx *F xx * * app i (a) Calculate the applied force required to move the bar to the right at a constant speed of 2.50 m/s. N (to the right) (b) At what rate is energy delivered to the resistor? W
Expert Solution
steps

Step by step

Solved in 3 steps with 3 images

Blurred answer
Knowledge Booster
Laws of electromagnetic induction
Learn more about
Need a deep-dive on the concept behind this application? Look no further. Learn more about this topic, physics and related others by exploring similar questions and additional content below.
Similar questions
Recommended textbooks for you
College Physics
College Physics
Physics
ISBN:
9781285737027
Author:
Raymond A. Serway, Chris Vuille
Publisher:
Cengage Learning
University Physics Volume 2
University Physics Volume 2
Physics
ISBN:
9781938168161
Author:
OpenStax
Publisher:
OpenStax
Physics for Scientists and Engineers, Technology …
Physics for Scientists and Engineers, Technology …
Physics
ISBN:
9781305116399
Author:
Raymond A. Serway, John W. Jewett
Publisher:
Cengage Learning
Principles of Physics: A Calculus-Based Text
Principles of Physics: A Calculus-Based Text
Physics
ISBN:
9781133104261
Author:
Raymond A. Serway, John W. Jewett
Publisher:
Cengage Learning
Physics for Scientists and Engineers with Modern …
Physics for Scientists and Engineers with Modern …
Physics
ISBN:
9781337553292
Author:
Raymond A. Serway, John W. Jewett
Publisher:
Cengage Learning
Physics for Scientists and Engineers
Physics for Scientists and Engineers
Physics
ISBN:
9781337553278
Author:
Raymond A. Serway, John W. Jewett
Publisher:
Cengage Learning